Regla de cuantización de Dirac

Primero recuerdo la regla de cuantización de Dirac, derivada bajo la hipótesis de que en algún lugar saldría una carga magnética: gramo q 4 π = norte 2 con norte un número natural

Me pregunto cómo se puede deducir la cuantización de la carga eléctrica. La cuantización del producto. gramo q ciertamente no es suficiente; ¿Qué más se exige?

La cuantización de gq ciertamente es suficiente, ya que si haces q pequeño, la unidad de g se eleva al infinito. Si hay cargas arbitrariamente pequeñas, el monopolo de carga más pequeño se carga infinitamente. Si hay un monopolo de carga fija, la unidad de carga es la inversa de la carga magnética. ¿Qué más hay que decir?
Su respuesta es suficiente en un universo con solo dos partículas: una carga magnética gramo y una carga electrica q . Pero si ahora consideras un universo más grande, con otras partículas, tengo la sensación de que sí, hay más que decir...
@Isaac: En ninguna parte de mi respuesta se supone que solo hay dos tipos de partículas. Por otro lado, si hemos establecido la cuantización de carga a través de dos tipos de partículas, tener más tipos de partículas alrededor significa más restricciones débiles (débilmente, porque pueden no ser restricciones nuevas), pero al menos no pueden deshacer las condiciones anteriores, solo apretarlos. Así que la cuantificación de carga permanece en su lugar.

Respuestas (4)

i) En primer lugar, la regla de cuantificación de Dirac

(1) q gramo 2 π     Z

para monopolos magnéticos se puede generalizar a la condición de cuantificación de Dirac-Zwanziger-Schwinger

(2) q 1 gramo 2 q 2 gramo 1 2 π     Z

para Dyons . (En un ligero mal uso de la terminología, a continuación también incluiremos partículas puramente cargadas eléctricamente y monopolos magnéticos puros en la definición de dion).

II) Deja Γ = { ( q , gramo ) } denote el conjunto de cargas eléctricas y magnéticas para dyons. Es natural pensar en Γ como un subconjunto del plano R 2 . El lado izquierdo de (2) tiene un significado geométrico como un área con signo dividida por dos vectores ( q 1 , gramo 1 ) y ( q 2 , gramo 2 ) .

III) Supongamos ahora que Γ { ( 0 , 0 ) } no está vacío, es decir, existe un dyon ( q 1 , gramo 1 ) ( 0 , 0 ) para empezar. que puntos ( q 2 , gramo 2 ) Γ de R 2 no entraría en conflicto con la condición (2)? La respuesta es un conjunto de rectas discretas equidistantes paralelas al vector ( q 1 , gramo 1 ) .

IV) Supongamos ahora que Γ contiene al menos dos vectores linealmente independientes ( q 1 , gramo 1 ) y ( q 2 , gramo 2 ) . que puntos ( q 3 , gramo 3 ) Γ de R 2 no entraría en conflicto con la condición (2)? La respuesta es una cuadrícula/retícula discreta de puntos de intersección, es decir, precisamente donde se encuentran los dos conjuntos correspondientes de líneas paralelas discretas equidistantes de la sección III. En otras palabras, los cargos están cuantificados.

V) Como caso especial, si existe al menos una partícula puramente cargada eléctricamente y al menos un monopolo magnético puro, estamos en la situación descrita en el apartado IV, por lo que las cargas deben estar cuantizadas.

Creo que esta es una pregunta abierta válida. Si resulta que no hay una sola carga magnética g, sino un continuo de carga magnética, entonces la condición de cuantización no sería una explicación suficiente para e.

Sin embargo, probar realmente si un continuo de carga magnética conduce o no a alguna contradicción requeriría, como mínimo, resolver un problema de n cuerpos con múltiples cargas magnéticas, lo cual no es un asunto trivial incluso para los profesionales en el campo. (o al menos un problema de 3 cuerpos con 2 cargas magnéticas a ver si surge una contradicción o reafirmación de la cuantización)

Si este tema ha sido abordado en la literatura, sería bueno que alguien con el conocimiento proporcione algunas citas como material de referencia para los interesados.

1) Suponga que existe una carga eléctrica mínima distinta de cero, q 0 . Por lo tanto, la carga magnética mínima es

gramo 0 = 2 π q 0 .

2) En segundo lugar, si la teoría conserva C y CP. Entonces el dyon ( q , gramo 0 ) automáticamente implica un conjugado dyon ( q , gramo 0 ) . Aplicando la condición de Dirac-Zwanziger (ver, en la respuesta de @Qmechanic) para estos dos dyons

2 q gramo 0 = 2 π norte ,

o,

q = norte 2 q 0 .

Así que tenemos dos posibilidades, la carga eléctrica q toma múltiplos enteros de q 0 , o toma múltiplos enteros impares de q 0 / 2 .

Intentaré responder desde una perspectiva matemática pura. La regla de cuantificación establece que para cualquier posible q y gramo , hay algo norte Z tal que q gramo = norte h .

Ahora considere el conjunto X = { norte Z + | q q + , gramo GRAMO +   s . t .   q gramo = norte h } , dónde q + contiene todas las posibles cargas positivas y GRAMO + contiene todas las posibles cargas magnéticas positivas (de monopolos magnéticos). Considere el elemento mínimo norte 0 en el conjunto X , entonces hay algo q 0 y gramo 0 que satisface q 0 gramo 0 = norte 0 h .

Tenga en cuenta que a menudo se supone que norte 0 = 1 , pero no es necesario en esta demostración.

Ahora considere algunos gramo gramo 0 . Después q 0 gramo = norte h para algunos norte . Ya que norte 0 es mínimo, tenemos norte > norte 0 . Ya que ambos norte y norte 0 son enteros, tenemos norte = pags norte 0 + r , 0 r < norte 0 , pags Z + . Si r 0 , después 0 < q 0 ( gramo pags gramo 0 ) = r h < norte h , contradiciendo que norte es mínimo Por lo tanto r = 0 . y gramo = pags gramo 0 , asi que gramo se cuantifica en la unidad de gramo 0 . Del mismo modo, podemos demostrar que q deben ser múltiplos de q 0 .

Hablando matemáticamente, las únicas suposiciones utilizadas anteriormente son que si q es un cargo válido, entonces q también es un cargo válido, y que si q 1 y q 2 son cargos válidos, entonces q 1 + q 2 es un cargo válido. lo mismo para gramo . Creo que estas suposiciones deberían ser bastante naturales dada la naturaleza física de q y gramo .